www.vorhilfe.de
Vorhilfe

Kostenlose Kommunikationsplattform für gegenseitige Hilfestellungen.
Hallo Gast!einloggen | registrieren ]
Startseite · Forum · Wissen · Kurse · Mitglieder · Team · Impressum
Forenbaum
^ Forenbaum
Status Englisch
  Status Grammatik
  Status Lektüre
  Status Korrekturlesen
  Status Übersetzung
  Status Sonstiges (Englisch)

Gezeigt werden alle Foren bis zur Tiefe 2

Navigation
 Startseite...
 Neuerdings beta neu
 Forum...
 vorwissen...
 vorkurse...
 Werkzeuge...
 Nachhilfevermittlung beta...
 Online-Spiele beta
 Suchen
 Verein...
 Impressum
Das Projekt
Server und Internetanbindung werden durch Spenden finanziert.
Organisiert wird das Projekt von unserem Koordinatorenteam.
Hunderte Mitglieder helfen ehrenamtlich in unseren moderierten Foren.
Anbieter der Seite ist der gemeinnützige Verein "Vorhilfe.de e.V.".
Partnerseiten
Weitere Fächer:

Open Source FunktionenplotterFunkyPlot: Kostenloser und quelloffener Funktionenplotter für Linux und andere Betriebssysteme
Forum "Uni-Analysis-Induktion" - Ungleichungskette
Ungleichungskette < Induktion < eindimensional < reell < Analysis < Hochschule < Mathe < Vorhilfe
Ansicht: [ geschachtelt ] | ^ Forum "Uni-Analysis-Induktion"  | ^^ Alle Foren  | ^ Forenbaum  | Materialien

Ungleichungskette: Frage (beantwortet)
Status: (Frage) beantwortet Status 
Datum: 15:02 Fr 23.10.2009
Autor: Blaub33r3

Aufgabe
Zeige durch vollständige Induktion für [mm] n\ge2 [/mm]
[mm] \summe_{k=1}^{n-1}<\bruch{n^4}{4} [/mm]

Hallo Leute,

A(n) = [mm] \summe_{k=1}^{n-1}k^3<\bruch{n^4}{4} [/mm]

A(0) = 1 < 4

Wenn A(n) und auch A(0) gilt, gilt auch A(n+1)

A(n) -> A(n+1)

[mm] \summe_{k=1}^{n}k^3=\summe_{k=1}^{n-1}k^3+n^3 <\bruch{n^4}{4}+n^3<\bruch{(n+1)^4}{4} [/mm]

Jetzt gilt es noch zuzeigen dass [mm] \bruch{n^4}{4}+n^3<\bruch{(n+1)^4}{4} [/mm] gilt, weil

[mm] \bruch{n^4+4n^3}{4} [/mm] < [mm] \bruch{n^4+4n^3+6n^2+4n}{4} [/mm]       q.e.d.

War der Beweis korrekt vollzogen?

Grüße Daniel

        
Bezug
Ungleichungskette: kleiner Fehler
Status: (Antwort) fertig Status 
Datum: 15:08 Fr 23.10.2009
Autor: Roadrunner

Hallo Daniel!


Prinzipiell ist Dein Nachweis korrekt.

Allerdings gilt:
[mm] $$(n+1)^4 [/mm] \ = \ [mm] n^4+4n^3+6n^2+4n+1$$ [/mm]

Gruß vom
Roadrunner


Bezug
                
Bezug
Ungleichungskette: Mitteilung
Status: (Mitteilung) Reaktion unnötig Status 
Datum: 15:11 Fr 23.10.2009
Autor: Blaub33r3

Danke dir!

Bezug
Ansicht: [ geschachtelt ] | ^ Forum "Uni-Analysis-Induktion"  | ^^ Alle Foren  | ^ Forenbaum  | Materialien


^ Seitenanfang ^
www.englischraum.de
[ Startseite | Forum | Wissen | Kurse | Mitglieder | Team | Impressum ]